I'm trying to show $$ \int_0^1 \frac{\ln x \cdot \ln(1+x)}{1-x}dx=-\frac{1}{4}\pi^2 \ln(2)+\zeta(3). $$ I am unsure how to approach this integral as I do not know how to use a power series representation for the integrand. I cannot use the generating function $$ \frac{\ln(1-x)}{1-x}=-\sum_{n=1}^\infty H_n x^n $$ since I do not have this in my integrand so it is not so easy to approach. Thanks for the help!
-
Why not using the same approach as for the previous problem where you had $(1+x)$ as denominator ? – Claude Leibovici Feb 25 '14 at 08:08
-
The argument in the log(1+x) differs from the denominator thus we cannot use the harmonic series generating function approach. @ClaudeLeibovici – Jeff Faraci Feb 25 '14 at 08:47
-
OK, but you can do Taylor expansion. – Claude Leibovici Feb 25 '14 at 09:12
-
@ClaudeLeibovici do you mean use the taylor series to obtain $$ \frac{\ln x}{1-x}=-\frac{\ln x}{x-1}=-\sum_{n=1}^\infty \frac{(-1)^{n+1}}{n} (x-1)^{n-1}? $$ So we get $$ \sum_{n=1}^\infty \frac{(-1)^{n}}{n}\int_0^1 (x-1)^{n-1}\ln(x+1) dx. $$ Are you saying to work with this? Thanks!!! – Jeff Faraci Feb 25 '14 at 14:25
-
See https://math.stackexchange.com/questions/3348463/prove-int-01-frac-ln-x-ln1x1-x-dx-zeta3-frac32-ln2-zeta2 – FDP Aug 25 '20 at 16:06
4 Answers
EDIT:
I modified my answer so that it no longer involves dealing with polylogarithms.
There is also a very nice evaluation HERE.
The generating function of the alternating harmonic numbers (i.e., $\bar{H}_{n} = \sum_{k=1}^{n} \frac{(-1)^{k-1}}{k})$ is $$\frac{\ln(1+x)}{1-x} = \sum_{n=1}^{\infty} \bar{H}_{n} x^{n}. $$
This can be derived using the Cauchy product.
(This was mentioned in a partial answer that was deleted shortly after it was posted.)
Using this generating function, we find that
$$ \begin{align} \int_{0}^{1} \frac{\ln (x) \ln(1+x)}{1-x} \, dx &= \int_{0}^{1} \ln(x) \sum_{n=1}^{\infty} \bar{H}_{n} \, x^{n} \, dx \\ &= \sum_{n=1}^{\infty} \bar{H}_{n} \int_{0}^{1} \ln(x) x^{n} \, dx \\ &= -\sum_{n=1}^{\infty} \frac{\bar{H}_{n}}{(n+1)^{2}} \\&= -\sum _{n=1}^{\infty} \frac{\bar{H}_{n+1}}{(n+1)^{2}} + \sum_{n=1}^{\infty} \frac{(-1)^{n}}{(n+1)^{3}} \\&= \left( -\sum_{n=1}^{\infty}\frac{\bar{H}_{n}}{n^{2}} +1 \right) + \left( \frac{ 3\zeta(3)}{4} -1 \right) \\ &= -\sum_{n=1}^{\infty}\frac{\bar{H}_{n}}{n^{2}} + \frac{3\zeta(3)}{4}. \end{align}$$
In general, $$2 \sum_{n=1}^{\infty} \frac{\bar{H}_{n}}{n^{q}} = 2 \zeta(q) \ln(2)-q \zeta(q+1) + 2 \eta(q+1) + \sum_{k=1}^{q} \eta(k)\eta(q-k+1) , $$ where $\eta(z)$ is the Dirichlet eta function.
(See Theorem 7.1 in this paper for details about how to derive this formula using contour integration.)
Therefore, $$\sum_{n=1}^{\infty} \frac{\bar{H}_{n}}{n^{2}} = \zeta(2) \ln(2) - \zeta(3) + \eta(3) + \eta(1) \eta(2) = \frac{\pi^{2}}{4} \ln(2) - \frac{\zeta(3)}{4}, $$ and
$$\int_{0}^{1} \frac{\ln (x) \ln(1+x)}{1-x} \, dx = \frac{\zeta(3)}{4} - \frac{\pi^{2}}{4} \log(2) + \frac{3\zeta(3)}{4} = - \frac{\pi^{2}}{4} \ln(2) + \zeta(3).$$

- 42,026
-
This is excellent. I figured out how to deal with the $\ln(-1)$ term, I hadn't realized there was another imaginary piece in the term which cancelled it out. Excellent solution, very nice, I will work through...Thanks again!! – Jeff Faraci Feb 25 '14 at 16:39
-
1It's messy and a bit confusing. You can derive the values of the dilogarithm and the trilogarithm at x=2 from functional equations. http://mathworld.wolfram.com/Dilogarithm.html http://mathworld.wolfram.com/Trilogarithm.html – Random Variable Feb 25 '14 at 16:45
-
in your second line. How did you change the sum over k from k=1 to k=0 without changing the $x^k$ term? Also, I think your dk should be dx. Thanks for the help, I am checking out these references you are giving me!!! – Jeff Faraci Feb 25 '14 at 16:47
-
-
I just have two questions..How did you go from $$ = \sum_{n=1}^{\infty} \frac{(-1)^{n-1}}{n} \sum_{k=0}^{\infty} \frac{-1}{(n+k+1)^{2}} = \sum_{n=1}^{\infty} \frac{(-1)^{n}}{n} \Big(\frac{\pi^{2}}{6} - \sum_{k=1}^{n} \frac{1}{k^{2}} \Big) $$ and how did you determine the constant of integration like that, do you just demand that at x=0 the sum must vanish since the left hand side does, thus we are only left with $C+2Li_3(1-x)=0$? That is the only way I can make sense of it! Thanks again... @RandomVariable – Jeff Faraci Feb 25 '14 at 21:06
-
1$$ \sum_{k=0}^{\infty} \frac{1}{(n+k+1)^{2}} = \sum_{k=1}^{\infty} \frac{1}{k^{2}} - \sum_{k=1}^{n} \frac{1}{k^{2}}$$
And assuming you meant $C + 2 \text{Li}_{3}(1-0) = 0$, that's exactly what I did.
– Random Variable Feb 25 '14 at 21:25 -
-
@RandomVariable Thanks! I have a question for you, how come this method doesn't work on the integral $$ \int_0^1 \frac{\ln(1-x)\ln x}{1+x}dx $$ cannot be done the same way as above it seems. When I try using this method I obtain $$ I=\sum_{n=1}^\infty \frac{1}{n} \sum_{k=0}^\infty \frac{(-1)^k}{(n+k+1)^2}. $$ But the sum of $1/n$ diverges. Thus I am not sure how to go about doing this integral, I would expect I could do it identical to above. Evaluating the trilog function and dilogarithm is very helpful, thanks!! – Jeff Faraci Feb 25 '14 at 22:20
-
1You have to be careful as to where those functional equations are valid. I don't know why it's not stated on that Wolfram web site. Wikipedia has additional information on the polylogarithm page. As for that other integral, that might be the most difficult of the six. You can't break up the sum like I did before due to convergence issues. You could switch the order of summation, but that probably won't help much. I'll try to come up with something later. – Random Variable Feb 25 '14 at 23:39
-
@RandomVariable . Yeah, I am looking into the polylogs on a few places online and some books regarding special functions. It sure seems like it...It is equal to $$\int_0^1 \frac{\ln (1-x)\ln x}{1+x} dx = \frac{1}{8}\big(-\pi^2\ln(4) +13\zeta(3)\big). $$ You are very good, thanks...talk to you later – Jeff Faraci Feb 26 '14 at 01:19
Use Feynman’s trick \begin{align} J(a) &= \int_0^1\frac{\ln x}{1+x}\left[ \frac{ \ln(1-ax)}x+\ln(1-x) \right]dx\\ J’(a) &=\int_0^1\frac{-\ln x\ dx}{(1+x)(1-ax)}= \frac{-1}{1+a}\int_0^1 \left(\frac{\ln x}{1+x}+ \frac{a\ln x}{1-ax} \right)dx \\ &= \frac{\zeta(2)}{2(1+a)}- \frac1{1+a}\int_0^a\frac{\ln y}{1-y}dy - \frac{\ln a\ln(1-a)}{1+a}\\ \end{align} Note that $$J(1) = \int_0^1 \frac{\ln x\ln(1-x)}{x}dx \overset{ibp} = \frac12\int_0^1 \frac{\ln^2x}{1-x}dx= \zeta(3) \tag1 $$ On the other hand \begin{align} J(1)& =J(0)+ \int_0^1 J’(a)da = \frac{\zeta(2)}2\ln2-\int_0^1 d[\ln(1+a)]\int_0^a \frac{\ln y}{1-y}dy \\ &\overset{ibp}= \frac{3}2\ln2\zeta(2)+ \int_0^1 \frac{\ln a \ln(1+a)}{1-a}da \tag2 \\\end{align}
Equate (1) and (2) to obtain
$$\int_0^1 \frac{\ln a \ln(1+a)}{1-a}da= -\frac{3}2\ln2\zeta(2)+\zeta(3)= -\frac{\pi^2}{4}\ln2+\zeta(3) $$

- 97,352
$\newcommand{\bbx}[1]{\,\bbox[15px,border:1px groove navy]{\displaystyle{#1}}\,} \newcommand{\braces}[1]{\left\lbrace\,{#1}\,\right\rbrace} \newcommand{\bracks}[1]{\left\lbrack\,{#1}\,\right\rbrack} \newcommand{\dd}{\mathrm{d}} \newcommand{\ds}[1]{\displaystyle{#1}} \newcommand{\expo}[1]{\,\mathrm{e}^{#1}\,} \newcommand{\ic}{\mathrm{i}} \newcommand{\mc}[1]{\mathcal{#1}} \newcommand{\mrm}[1]{\mathrm{#1}} \newcommand{\pars}[1]{\left(\,{#1}\,\right)} \newcommand{\partiald}[3][]{\frac{\partial^{#1} #2}{\partial #3^{#1}}} \newcommand{\root}[2][]{\,\sqrt[#1]{\,{#2}\,}\,} \newcommand{\totald}[3][]{\frac{\mathrm{d}^{#1} #2}{\mathrm{d} #3^{#1}}} \newcommand{\verts}[1]{\left\vert\,{#1}\,\right\vert}$ $\ds{\bbox[5px,#ffd]{\int_{0}^{1}{\ln\pars{x}\ln\pars{1 + x} \over 1 - x}\,\dd x = -\,{1 \over 4}\,\pi^{2}\ln\pars{2} + \zeta\pars{3}} \approx -0.5082:\ {\Large ?}}$.
\begin{align} &\bbox[5px,#ffd]{% \int_{0}^{1}{\ln\pars{x}\ln\pars{1 + x} \over 1 - x}\,\dd x} \\[5mm] = &\ {1 \over 2}\int_{0}^{1}{\ln^{2}\pars{x} \over 1 - x}\,\dd x + {1 \over 2}\int_{0}^{1}{\ln^{2}\pars{1 + x} - \ln^{2}\pars{2}\over 1 - x}\,\dd x \\[2mm] & \!\!\!\!\! -\,{1 \over 2}\int_{0}^{1} \bracks{\ln^{2}\pars{x \over 1 + x} - \ln^{2}\pars{2}}\,{\dd x \over 1 - x} \end{align} In the second integral I'll make the change $\ds{{1 + x \over 2} \mapsto x}$ while I'll set $\ds{{x \over x + 1} \mapsto x}$ in the last one: \begin{align} &\bbox[5px,#ffd]{% \int_{0}^{1}{\ln\pars{x}\ln\pars{1 + x} \over 1 - x}\,\dd x} \\[5mm] = &\ {1 \over 2}\ \overbrace{\int_{0}^{1}{\ln^{2}\pars{x} \over 1 - x}\,\dd x} ^{\ds{I_{1}}}\ +\ {1 \over 2}\ \overbrace{\int_{1/2}^{1} {\ln^{2}\pars{2x} - \ln^{2}\pars{2}\over 1 - x}\,\dd x}^{\ds{I_{2}}} \\[2mm] & \!\!\!\!\! -\,{1 \over 2}\bracks{% \underbrace{\int_{0}^{1}{\ln^{2}\pars{x/2} - \ln^{2}\pars{2} \over 1 - x}\,\dd x}_{\ds{I_{3}}}\ -\ \underbrace{\int_{0}^{1/2}{\ln^{2}\pars{x} - \ln^{2}\pars{2} \over 1 - x}\,\dd x}_{\ds{I_{4}}}}\label{1}\tag{1} \end{align} In the last line first integral I did the additional scaling $\ds{x \mapsto {x \over 2}}$.
In (\ref{1}), all the integrals are reduced to simple ones by integrating twice by parts. Namely, \begin{equation} \left\{\begin{array}{rcl} \ds{I_{1}} & \ds{=} & \ds{\phantom{-\,\,}2\zeta\pars{3}} \\[2mm] \ds{I_{2}} & \ds{=} & \ds{\phantom{-\,\,}{2\ln^{3}\pars{2} \over 3} - {\pi^{2}\ln\pars{2} \over 6} + {\zeta\pars{3} \over 4}} \\[2mm] \ds{I_{3}} & \ds{=} & \ds{\phantom{-\,\,}{\pi^{2}\ln\pars{2} \over 3} + 2\zeta\pars{3}} \\[2mm] \ds{I_{4}} & \ds{=} & \ds{-\,{2\ln^{3}\pars{2} \over 3} + {7\zeta\pars{3} \over 4}} \end{array}\right. \end{equation} Then, \begin{align} &\bbox[5px,#ffd]{% \int_{0}^{1}{\ln\pars{x}\ln\pars{1 + x} \over 1 - x}\,\dd x} = {I_{1} + I_{2} - I_{3} + I_{4} \over 2} \\[5mm] = &\ \bbx{-\,{1 \over 4}\,\pi^{2}\ln\pars{2} + \zeta\pars{3}} \approx -0.5082 \\ & \end{align}

- 89,464
\begin{align}J&=\int_{0}^{1} \frac{\ln(x) \ln (1+x)}{1-x} \, \mathrm{d} x\\&\overset{\text{IBP}}=\left[\left(\int_0^x \frac{\ln t}{1-t}dt\right)\ln(1+x)\right]_0^1-\int_0^1 \frac{1}{1+x}\left(\int_0^x \frac{\ln t}{1-t}dt\right)dx\\&=-\zeta(2)\ln 2-\int_0^1\int_0^1 \frac{x\ln(tx)}{(1+x)(1-tx)}dtdx\\ &=-\zeta(2)\ln 2-\int_0^1\int_0^1 \frac{\ln(tx)}{(1+t)(1-tx)}dtdx+\int_0^1\int_0^1 \frac{\ln(tx)}{(1+x)(1+t)}dtdx\\ &\overset{u(x)=tx}=-\zeta(2)\ln 2-\int_0^1 \frac{1}{t(1+t)}\left(\int_0^t \frac{\ln u}{1-u}du\right)dt+2\ln 2\int_0^1 \frac{\ln x}{1+x}dx\\ &\overset{\text{IBP}}=-\zeta(2)\ln 2-\left[\ln\left(\frac{t}{1+t}\right)\left(\int_0^t \frac{\ln u}{1-u}du\right)\right]_0^1+\int_0^1 \frac{\ln\left(\frac{t}{1+t}\right)\ln t}{1-t}dt+\\&2\ln 2\int_0^1 \frac{\ln x}{1+x}dx\\ &=-2\zeta(2)\ln 2+\int_0^1 \frac{\ln^2 t}{1-t}dt-J+2\ln 2\int_0^1 \frac{\ln x}{1+x}dx\\ &=-2\zeta(2)\ln 2+2\zeta(3)-J+2\ln 2 \times -\frac{1}{2}\zeta(2)\\ &=-3\zeta(2)\ln 2+2\zeta(3)-J\\ &=-3\times \frac{\pi^2}{6}\ln^ 2+2\zeta(3)-J\\ J&=\boxed{\zeta(3)-\frac{1}{4}\pi^2\ln 2} \end{align}
NB: \begin{align}\int_0^1 \frac{\ln x}{1+x}dx&=\int_0^1 \frac{\ln x}{1-x}dx-\int_0^1 \frac{2t\ln t}{1-t^2}dt\\ &\overset{x=t^2}=\int_0^1 \frac{\ln x}{1-x}dx-\frac{1}{2}\int_0^1 \frac{\ln x}{1-x}dx\\ &=\frac{1}{2}\int_0^1 \frac{\ln x}{1-x}dx\\ &=-\frac{1}{2}\zeta(2)\\ \end{align} Moreover, i assume that: \begin{align}\int_0^1 \frac{\ln x}{1-x}dx&=-\zeta(2)\\ \zeta(2)&=\frac{1}{6}\pi^2\\ \int_0^1 \frac{\ln x}{1-x}dx&=2\zeta(3)\\ \end{align}

- 13,647